LSAT and Law School Admissions Forum

Get expert LSAT preparation and law school admissions advice from PowerScore Test Preparation.

 ellenb
  • Posts: 260
  • Joined: Oct 22, 2012
|
#9677
Dear Powerscore,

For this question, why answer C is not the right one and answer B is the right one?
I thought to eliminate C because it stated by a majority of their constituents. The word majority made me eliminate it, since I cannot prove it.

Thanks

Ellen
 Steve Stein
PowerScore Staff
  • PowerScore Staff
  • Posts: 1153
  • Joined: Apr 11, 2011
|
#9681
Hi Ellen,

That's exactly right; the author in this case made no claims about majority support of particular activities, so answer choice C would not help to support the columnist's argument.

I hope that's helpful!

~Steve
 simshrops
  • Posts: 1
  • Joined: Feb 23, 2020
|
#75902
Hi there,

My original answer was C because I didn't pick up on the fact that the author made no claims about majority support of particular activities and that saying so cannot be proven true. I now understand why B is correct because it proves that premise -> conclusion. Just so I have a full understanding of the causal logic in the stimulus, would it be correct to say that the conditional chain is:

The representatives support the arts with tax dollars -> support for arts is warranted -> taxpayers were not treated unfairly

Therefore, would it be correct to say that the chain above would support the premise -> conclusion argument by proving that if the representatives support the arts with tax dollars, then the support for the arts is warranted and taxpayers were not treated unfairly?
 Adam Tyson
PowerScore Staff
  • PowerScore Staff
  • Posts: 5153
  • Joined: Apr 14, 2011
|
#75988
This argument didn't really jump out at me as conditional, simshrops, and it feels like a little too much unnecessary work to put it in that framework. There wasn't an if...then structure to any of it that I noticed (although there is always something of a conditional element to any argument, in that the author believes "if my premises are true, then my conclusion must follow from them" ). Instead, I would say the argument went like this:

Premise: Representatives were within their rights to fund art in general
Conclusion: Taxpayers were not treated unfairly just because they disliked some of the art

Prephrase to strengthen that claim: When representative do things they have a right to do, then nobody has been treated unfairly, even if they don't like it all.

The stuff in answer B about an activity being "warranted" can be interpreted as meaning that it is okay, justifiable, fair. The "particular activity" is one or another offensive or otherwise displeasing artwork. The "general" activity is funding art.

Your conditional approach is accurate, and if that proved helpful then more power to you! But I would be careful about inserting conditional analyses into arguments where they don't need to go, for fear that you might end up missing or misinterpreting something critical. For example, some causal arguments can be improperly translated into conditional arguments, and the analysis of them becomes flawed because of important differences between the two.
 ericj_williams
  • Posts: 63
  • Joined: Jan 19, 2020
|
#85417
Adam Tyson wrote: Fri Jun 05, 2020 4:24 pm This argument didn't really jump out at me as conditional, simshrops, and it feels like a little too much unnecessary work to put it in that framework. There wasn't an if...then structure to any of it that I noticed (although there is always something of a conditional element to any argument, in that the author believes "if my premises are true, then my conclusion must follow from them" ). Instead, I would say the argument went like this:

Premise: Representatives were within their rights to fund art in general
Conclusion: Taxpayers were not treated unfairly just because they disliked some of the art

Prephrase to strengthen that claim: When representative do things they have a right to do, then nobody has been treated unfairly, even if they don't like it all.

The stuff in answer B about an activity being "warranted" can be interpreted as meaning that it is okay, justifiable, fair. The "particular activity" is one or another offensive or otherwise displeasing artwork. The "general" activity is funding art.

Your conditional approach is accurate, and if that proved helpful then more power to you! But I would be careful about inserting conditional analyses into arguments where they don't need to go, for fear that you might end up missing or misinterpreting something critical. For example, some causal arguments can be improperly translated into conditional arguments, and the analysis of them becomes flawed because of important differences between the two.
So I picked B.

My issue with this answer is that it goes from not being treated unjustly to legitimate.

Basically, I feel that one has to equate not being treated unjustly with being legitimate.

If not, you could say that being treated justly does not in itself provide for legitimate funding, as in a legitimate funding could have additional requirements beyond mere just treatment.

What are your thoughts on that term shift?
 Jeremy Press
PowerScore Staff
  • PowerScore Staff
  • Posts: 1000
  • Joined: Jun 12, 2017
|
#85529
Hi Eric,

Answer choice B's reference to "elected representatives who legitimately fund [an] activity" is actually intended to cover the statement in the premises that the "elected government representatives were within their rights to vote to support the arts with tax dollars." If they were "within their right" to fund the arts, then their funding of the arts was "legitimate." The only part of answer choice B that needs to cover the conclusion's statement that no one has been treated unjustly is the part that talks about the funding being "warranted" (see Adam's discussion of fairness for that one).

Side note: where a Strengthen-Principle answer is phrased as a conditional statement, what you want to see is that the sufficient condition of the principle covers the premise(s) of the stimulus argument. And you also want to see that the necessary condition of the principle covers the conclusion of the stimulus argument. This is because the way such a principle strengthens the argument is by showing that "If this set of premises is true, then the conclusion is necessarily true."

I hope this helps!
User avatar
 JonathanLee73
  • Posts: 1
  • Joined: Jul 30, 2023
|
#104634
Why is C incorrect? I understand in the stimulus the author never claimed anything about a majority. But the powerscore Bible says that “Answer choices are accepted as given, even if they include “new information”. Just because a fact or idea is not mentioned in the stimulus is not grounds for dismissing an answer choice
 Robert Carroll
PowerScore Staff
  • PowerScore Staff
  • Posts: 1787
  • Joined: Dec 06, 2013
|
#104836
JonathanLee73,

The problem is you're bringing up information outside of the answer choice AND the stimulus. There's nothing in the stimulus that has majority support. So answer choice (C) is not supporting anything in the stimulus.

Imagine an answer said "Any legislation that is proposed by Senator Casey is justified." Well, what's been proposed by Senator Casey? We have no idea - nothing in the stimulus talks about Senator Casey, and my hypothetical answer never connects Senator Casey with this particular government act. We'd have, then, that certain things which may or may not be the kinds of things that happened in the stimulus are justified. That's doing no good. Answer choice (C) has exactly the same problem.

Robert Carroll

Get the most out of your LSAT Prep Plus subscription.

Analyze and track your performance with our Testing and Analytics Package.